Đến nội dung

hanguyen445

hanguyen445

Đăng ký: 04-02-2015
Offline Đăng nhập: 24-03-2024 - 15:12
***--

#744269 với $d$ là số nguyên dương, CMR:không tồn tại các số nguyên $x...

Gửi bởi hanguyen445 trong 20-03-2024 - 16:28

1/  tìm các số nguyên $x,y,z$ sao cho $x+y+z=0$ và $xy+yz+zx+3=0$.

2/  Cho $t$ là số nguyên, chứng minh rằng nếu tồn tại các số nguyên $x,y,z$ khác $0$ sao cho $x+y+z=0$ và $xy+yz+zx+4t=0$ thì cũng tồn tại các số nguyên $x',y',z'$ khác $0$ sao cho $x'+y'+z'=0$ và $x'y'+y'z'+z'x'+t=0$.

3/ với $d$ là số nguyên dương, chứng minh rằng không tồn tại các số nguyên $x,y,z$ khác $0$ sao cho $x+y+z=0$ và $xy+yz+zx+2^{d}=0$

Bài 2. Ta có

$xy+yz+zx=-4t \Rightarrow  xy+yz+zx\equiv 0(mod 4)$; $x^2+y^2+z^2=-2(xy+yz+zx)\Rightarrow x^2+y^2+z^2\equiv 0(mod 4) (*)$

Chú ý rằng $\forall t\in R $ thì $t^2\equiv 0;1(mod 4)$, kết hợp với (*) ta suy ra $x^2,y^2,z^2\equiv 0(mod 4)\Rightarrow x;y;z\equiv 0(mod 2) $

Đặt $(x;y;z)=(2x';2y';2z')\ne (0;0;0)$ (***), thay ngược lại vào đẳng thức $xy+yz+zx+4t=0$ thì ta có 

$$x'+y'+z'+t=0$$

Từ (***) hiển nhiên $(x';y';z')\ne (0;0;0)$, do đó phép chứng minh hoàn tất.




#744268 với $d$ là số nguyên dương, CMR:không tồn tại các số nguyên $x...

Gửi bởi hanguyen445 trong 20-03-2024 - 16:19

1/  tìm các số nguyên $x,y,z$ sao cho $x+y+z=0$ và $xy+yz+zx+3=0$.

2/  Cho $t$ là số nguyên, chứng minh rằng nếu tồn tại các số nguyên $x,y,z$ khác $0$ sao cho $x+y+z=0$ và $xy+yz+zx+4t=0$ thì cũng tồn tại các số nguyên $x',y',z'$ khác $0$ sao cho $x'+y'+z'=0$ và $x'y'+y'z'+z'x'+t=0$.

3/ với $d$ là số nguyên dương, chứng minh rằng không tồn tại các số nguyên $x,y,z$ khác $0$ sao cho $x+y+z=0$ và $xy+yz+zx+2^{d}=0$

Bài 1. Do vai trò x, y, z bình đẳng và trong ba số x,y,z luôn có hai số cùng dấu. Không giảm tổng quát giả sử $xy\ge 0$.

Ta có $xy+3=(x+y)^2\Leftrightarrow x^2+y^2+xy=3 (*)$. Nếu trong (*) có ít nhất một số bằng 0, hiển nhiên vô lý. Do đó $x^2+y^2+xy\ge 3$, 

như vậy (*) xảy ra khi và chỉ khi (x;y) nhận các cặp là $(1;1); (-1;-1)$, thay ngược lại tương ứng z nhận {-2;2}.

Vậy bộ (x;y;z) cần tìm là (1;1;-2) và (-1;-1;2)




#742880 ​$\frac{a}{b(b+2c)^2}+\frac{b}...

Gửi bởi hanguyen445 trong 03-01-2024 - 15:03

Cho $a; b; c$ là các số thực dương thoả mãn $(a+b)(b+c)(c+a)=1$. Chứng minh rằng: 
$\frac{a}{b(b+2c)^2}+\frac{b}{c(c+2a)^2}+\frac{c}{a(a+2b)^2}\geq \frac{4}{3}$

Ta có $ \frac{a}{b(b+2c)^2}=\frac{(\frac{a}{b+2c})^2}{ab}$. Biến đổi tương tự với các số hạng còn lại trong biểu thức vế trái và sử dụng BĐT cauchy-schwarz ta có $$VT\ge (\frac{a}{b+2c}+\frac{b}{c+2a}+\frac{c}{a+2b})^2.\frac{1}{ab+bc+ac}$$

Lại có $\frac{a}{b+2c}+\frac{b}{c+2a}+\frac{c}{a+2b}\ge \frac{(a+b+c)^2}{3(ab+bc+ac)}$. Khi đó

$$VT\ge\frac{(a+b+c)^4}{9(ab+bc+ac)^3}\ge\frac{1}{ab+bc+ac}$$

Có $1=(a+b)(b+c)(c+a)=(a+b+c)(ab+bc+ac)-abc\ge\frac{8}{9}. (ab+bc+ac)(a+b+c)\ge\frac{8}{9}(ab+bc+ac)\sqrt{3(ab+bc+ac)}$.

Hay $ab+bc+ac\le\frac{3}{4}$. Do đó ta suy ra được $VT\ge\frac{4}{3}$. Phép chứng minh hoàn tất.


  • MPU yêu thích


#742561 $\frac{xy}{z}+\frac{yz}{x...

Gửi bởi hanguyen445 trong 18-12-2023 - 15:30

BÀI TOÁN 8. Với $a, b, c,  m, n, p, x, y, z$ là các số thực dương chứng minh rằng $$(a^3+b^3+c^3) (m^3+n^3+p^3) (x^3+y^3+z^3) \geq (amx+bny+cpz)^3$$

Đây là Bất đẳng thức Holder cho 3 dãy. Diễn đàn đã có chứng minh tại đây, tuy nhiên thì bạn nào chưa biết bất đẳng thức này có thể thử tự tìm cách giải. Gợi ý: sử dụng AM-GM. 

 

BÀI TOÁN 9. Với $a, b, c$ là các số thực dương chứng minh rằng 

b) $P=\frac{a}{\sqrt{2b^2+2c^2-a^2}}+\frac{b}{\sqrt{2a^2+2c^2-b^2}}+\frac{c}{\sqrt{2a^2+2b^2-c^2}} \geq \sqrt{3}$

Bài toán 9b: Sử dụng BĐT cauchy-schwarz ta có

$P=\sum\frac{a^2}{\sqrt{a(2ab^2+2ac^2-a^3)}}\ge\frac{(a+b+c)^2}{\sqrt{(a+b+c)T}}$ với $T=\sum (2ab^2+2ac^2-a^3)$

Lại có $T=\sum 2(a+b+c)(ab+bc+ac)-a^3-b^3-c^3-6abc$ và sử dụng BĐT schur ta có $ a^3+b^3+c^3+3abc\ge\sum ab(a+b)$

Do đó suy ra $T\le 2(a+b+c)(ab+bc+ac)-(a+b+c)(ab+bc+ac)=(a+b+c)(ab+bc+ac)\le\frac{(a+b+c)^3}{3}$.

Như vậy thì $P\ge \sqrt{3}$. Phép chứng minh hoàn tất.

Chú ý: Để bất đẳng thức xảy ra thì ta cần thêm điều kiện $Min (b^2+c^2-a^2/2; a^2+c^2-b^2/2; a^2+b^2-c^2/2)>0$




#742560 $\frac{xy}{z}+\frac{yz}{x...

Gửi bởi hanguyen445 trong 18-12-2023 - 15:04

BÀI TOÁN 8. Với $a, b, c,  m, n, p, x, y, z$ là các số thực dương chứng minh rằng $$(a^3+b^3+c^3) (m^3+n^3+p^3) (x^3+y^3+z^3) \geq (amx+bny+cpz)^3$$

Đây là Bất đẳng thức Holder cho 3 dãy. Diễn đàn đã có chứng minh tại đây, tuy nhiên thì bạn nào chưa biết bất đẳng thức này có thể thử tự tìm cách giải. Gợi ý: sử dụng AM-GM. 

 

BÀI TOÁN 9. Với $a, b, c$ là các số thực dương chứng minh rằng 

a) $\frac{a}{\sqrt{a^2+8bc}}+\frac{b}{\sqrt{b^2+8ac}}+\frac{c}{\sqrt{c^2+8ab}} \geq 1$

b) $\frac{a}{\sqrt{2b^2+2c^2-a^2}}+\frac{b}{\sqrt{2a^2+2c^2-b^2}}+\frac{c}{\sqrt{2a^2+2b^2-c^2}} \geq \sqrt{3}$

Bài toán 9 a: 

Đặt $P=\sum\frac{a}{\sqrt{a^2+8bc}}$. Sử BĐT cauchy-schwarz ta có

$$P=\sum \frac{a^2}{a\sqrt{a^2+8bc}}\ge\frac{(a+b+c)^2}{\sqrt{a(a^3+8abc)}}\ge\frac{(a+b+c)^2}{\sqrt{(a+b+c)(\sum(a^3+8abc)}}$$

Lại có

 

$\sum a(a^2+8bc)=a^3+b^3+c^3+24abc$

 

$=(a+b+c)(a^2+b^2+c^2-ab-bc-ac)+27abc$

 

$\le (a+b+c)(a^2+b^2+c^2-ab-bc-ac)+3(a+b+c)(ab+bc+ac)=(a+b+c)^3$.

 

Do đó ta suy ra được $P\ge\frac{(a+b+c)^2}{(a+b+c)^2}=1$. Hoàn tất chứng minh




#742514 $\frac{xy}{z}+\frac{yz}{x...

Gửi bởi hanguyen445 trong 15-12-2023 - 00:18

BÀI TOÁN 7. Tìm giá trị nhỏ nhất của biểu thức $$S=\frac{3a}{b+c}+\frac{4b}{a+c}+\frac{5c}{a+b}.$$

Biến đổi S và kết hợp với sử dụng BĐT Cauchy-Shwarz ta có

$S+12=(a+b+c)(\frac{3}{b+c}+\frac{4}{a+c}+\frac{5}{a+b})$

$\geq (a+b+c)\frac{(2+\sqrt{3}+\sqrt{5})^2}{2(a+b+c)}=\frac{(2+\sqrt{3}+\sqrt{5})^2}{2}$ 

Hay $S\ge\frac{(2+\sqrt{3}+\sqrt{5})^2}{2}-12$. Do đó $Min S =\frac{(2+\sqrt{3}+\sqrt{5})^2}{2}-12$. 




#742486 Chứng minh bất đẳng thức $\sum\frac{1}{a^2+2bc...

Gửi bởi hanguyen445 trong 13-12-2023 - 00:13

Bài toán 1: Cho các số thực dương $a,b,c$ thỏa mãn $a+b+c=1$. Chứng minh rằng 

$$\frac{1}{a^2+2bc}+\frac{1}{b^2+2ac}+\frac{1}{c^2+2ab}\ge\frac{1}{\sqrt{a^2-ab+b^2}}+\frac{1}{\sqrt{b^2-bc+c^2}}+\frac{1}{\sqrt{c^2-ca+a^2}}$$

 

Bài toán 2: Cho các số thực dương $a,b,c$ thỏa mãn $abc=1$. Chứng minh rằng

$$\frac{a^3+1}{\sqrt{a^4+b+c}}+\frac{b^3+1}{\sqrt{b^4+a+c}}+\frac{c^3+1}{\sqrt{c^4+a+b}}\ge 2\sqrt{ab+bc+ac}$$




#742469 $\frac{xy}{z}+\frac{yz}{x...

Gửi bởi hanguyen445 trong 11-12-2023 - 17:11

 

Thấy chỗ này sôi nổi quá nên cho mình ké bài của mình ạ:vv
Chứng minh bất đẳng thức sau $$\frac{a}{(b+2c)^2\sqrt{b^2+3}}+\frac{b}{(c+2a)^2\sqrt{c^2+3}}+\frac{c}{(a+2b)^2\sqrt{a^2+3}}\geq\frac{1}{2(a+b+c)}$$
2) Với $a,b,c$ là các số thực dương và $ab+bc+ca=3$
 

 

Trường hợp số 2: Biến đổi  và kết hợp với sử dụng BĐT cauchy-schwarz ta có
 
$$VT=\sum (\frac{a}{b+2c})^2.\frac{1}{a\sqrt{b^2+3}}\ge (\sum\frac{a}{b+2c})^2.\frac{1}{\sum a\sqrt{b^2+3}}$$
 
 
$$\sum\frac{a}{b+2c}=\sum\frac{a^2}{ab+2ac}\ge\frac{(a+b+c)^2}{3(ab+bc+ac)}=\frac{3}{ab+bc+ac}$$
 
$$\sum a\sqrt{b^2+3}\le\sqrt{(a^2+b^2+c^2)(a^2+b^2+c^2+9)}$$
 
Do đó ta suy ra được $VT\ge\frac{(a+b+c)^4}{(ab+bc+ac)^2 \sqrt{(a^2+b^2+c^2)(a^2+b^2+c^2+9)}}$. 
Đặt $M=(ab+bc+ac)^3(a^2+b^2+c^2)(a^2+b^2+c^2+9)$, hơn nữa ta lại có:
 
$$4^3(ab+bc+ac)^3.4(a^2+b^2+c^2)(a^2+b^2+c^2+9)\le (\frac{12(ab+bc+ac)+5(a^2+b^2+c^2+9)}{5})^5$$
 
Có  $12(ab+bc+ac)+5(a^2+b^2+c^2)+9\le \dfrac{20}{3}(a+b+c)^2$. Suy ra $M\le\frac{4}{3^5}(a+b+c)^{10}$
Suy ra $ (ab+bc+ac)^2\sqrt{(a^2+b^2+c^2)(a^2+b^2+c^2+9)}=\sqrt{(ab+bc+ac).M}\le\frac{2}{9}(a+b+c)^5$
 
Do đó ta suy ra được $VT\ge\frac{(a+b+c)^4}{2(a+b+c)^5}$. Hay $VT\ge\frac{1}{2(a+b+c)}$.
Như vậy phép chứng minh hoàn tất.



#742468 $\frac{xy}{z}+\frac{yz}{x...

Gửi bởi hanguyen445 trong 11-12-2023 - 16:39

 

Thấy chỗ này sôi nổi quá nên cho mình ké bài của mình ạ:vv
Chứng minh bất đẳng thức sau $$\frac{a}{(b+2c)^2\sqrt{b^2+3}}+\frac{b}{(c+2a)^2\sqrt{c^2+3}}+\frac{c}{(a+2b)^2\sqrt{a^2+3}}\geq\frac{1}{2(a+b+c)}$$
3) Với $a,b,c$ là các số thực dương và $a+b+c=3$

 

Trường hợp số 3: Biến đổi  và kết hợp với sử dụng BĐT cauchy-schwarz ta có

$$VT=\sum (\frac{a}{b+2c})^2.\frac{1}{a\sqrt{b^2+3}}\ge (\sum\frac{a}{b+2c})^2.\frac{1}{\sum a\sqrt{b^2+3}}$$

$$\sum\frac{a}{b+2c}=\sum\frac{a^2}{ab+2ac}\ge\frac{(a+b+c)^2}{3(ab+bc+ac)}=\frac{3}{ab+bc+ac}$$

$$\sum a\sqrt{b^2+3}\le\sqrt{(a^2+b^2+c^2)(a^2+b^2+c^2+9)}$$

Do đó ta suy ra được $VT\ge\frac{9}{(ab+bc+ac)^2 \sqrt{(a^2+b^2+c^2)(a^2+b^2+c^2+9)}}$. Hơn nữa ta lại có

$$M=4^4(ab+bc+ac)^4.4(a^2+b^2+c^2)(a^2+b^2+c^2+9)\le (\frac{16(ab+bc+ac)+5(a^2+b^2+c^2+9)}{6})^6$$

Ta lại có $16(ab+bc+ac)+5(a^2+b^2+c^2)+9\le 7(a+b+c)^2+9=72$.

Suy ra $(ab+bc+ac)^2 \sqrt{(a^2+b^2+c^2)(a^2+b^2+c^2+9)}\le 54$.

 

Như vậy thì $VT\ge\frac{9}{54}=\frac{1}{6}=\frac{1}{2(a+b+c)}$. Do đó phép chứng minh hoàn tất.




#742454 $\frac{xy}{z}+\frac{yz}{x...

Gửi bởi hanguyen445 trong 10-12-2023 - 15:41

BÀI TOÁN 6. Giả sử $x, y, z \geq 1$ và $\frac{1}{x}+\frac{1}{y}+\frac{1}{z}=2$. Chứng minh $$\sqrt{x+y+z} \geq \sqrt{x-1}+\sqrt{y-1}+\sqrt{z-1}.$$      

Biến đổi và sử dụng BĐT cauchy-schwarz ta có  $\frac{1}{x}=2\Leftrightarrow \sum\frac{x-1}{x}=1\ge\dfrac{ (\sum\sqrt{x-1})^2}{x+y+z}$

Hay $\sqrt{x+y+z}\ge\sum\sqrt{x-1}$. Do đó phép chứng minh hoàn tất.




#742335 $\frac{a+b+c}{3}-\sqrt[3]{abc}...

Gửi bởi hanguyen445 trong 02-12-2023 - 23:28

cho a,b,c là các số thực dương. CMR:
$\frac{a+b+c}{3}-\sqrt[3]{abc}\leq max {(\sqrt{a}-\sqrt{b})^{2},(\sqrt{b}-\sqrt{c})^{2},(\sqrt{c}-\sqrt{a})^{2}}$

                                                                                    (đề chọn đội tuyển dự thi Toán quốc tế, Mỹ năm 2000)

Do bài toán đối xứng với ba biến $a, b, c$, không giảm tổng quát giả sử $a\ge b\ge c$.

Lúc này ta suy ra được $(\sqrt{a}-\sqrt{c})^2=\text{Max}( (\sqrt{a}-\sqrt{c})^2;(\sqrt{a}-\sqrt{b})^2;(\sqrt{b}-\sqrt{c})^2)$

Do đó BĐT cần chứng minh tương đương

 

$$\frac{a+b+c}{3}-\sqrt[3]{abc}\le (\sqrt{c}-\sqrt{a})^{2}$$

 

Đặt $(x;y;z)=(\sqrt[6]{a},\sqrt[6]{b},\sqrt[6]{c})$ và hiển nhiên $x\ge y\ge z\ge 0$,  khi đó bất đẳng thức viết lại thành 

$$x^6+y^6+z^6-3(xyz)^2\le 3(x^3-z^3)^2$$

Ta có:

 

$x^6+y^6+z^6-3(xyz)^2=(x^2+y^2+z^2)[(x^2-z^2)^2+(y^2-x^2)(y^2-z^2)$

 

$3(x^3-z^3)^2=3(x-z)^2(x^2+xz+z^2)^2$

 

Hiển nhiên $(y^2-x^2)(y^2-z^2)\le 0$ với mọi $x\ge y\ge z$. Như vậy bất đẳng thức đúng nếu ta chứng minh được

$$(x^2+y^2+z^2)(x+z)^2\le 3(x^2+xz+z^2)^2(**)$$

Ta có $(x^2+xz+z^2)\ge\frac{3}{4} (x+z)^2$. Do đó ta suy ra được $VP(**)\ge\frac{9}{4}(x+z)^2(x^2+xz+z^2)$, nên BĐT đúng nếu

$$9(x^2+xz+z^2)\ge 4(x^2+y^2+z^2) (3*)$$

Hiển nhiên: $9x^2>8x^2>4(x^2+y^2)$ và $9(xz+z^2)\ge 4z^2$. Cộng vế ta suy ra được (3*) được chứng minh.

 

Vậy phép chứng minh hoàn tất.




#742322 $\frac{xy}{z}+\frac{yz}{x...

Gửi bởi hanguyen445 trong 02-12-2023 - 10:04

 

Thấy chỗ này sôi nổi quá nên cho mình ké bài của mình ạ:vv
Chứng minh bất đẳng thức sau $$\frac{a}{(b+2c)^2\sqrt{b^2+3}}+\frac{b}{(c+2a)^2\sqrt{c^2+3}}+\frac{c}{(a+2b)^2\sqrt{a^2+3}}\geq\frac{1}{2(a+b+c)}$$
1) Với $a,b,c$ là các số thực dương và $a^2+b^2+c^2=3$
2) Với $a,b,c$ là các số thực dương và $ab+bc+ca=3$
3) Với $a,b,c$ là các số thực dương và $a+b+c=3$

 

Trường hợp 1:

Ta có $\frac{a}{(b+2c)^2\sqrt{b^2+3}}=\dfrac{(\dfrac{a}{b+2c})^2}{a\sqrt{b^2+3}}$, biến đổi tương tự với các số hạng còn lại và sử dụng BĐT cauchy-schwarz cho vế trái. Ta có:

$VT\ge P^2.\frac{1}{Q}$ với:

$$P=\frac{a}{b+2c}+\frac{b}{c+2a}+\frac{c}{a+2b}\ge \frac{(a+b+c)^2}{3(ab+bc+ac)}\ge 1 (**)$$

$$Q=a\sqrt{b^2+3}+b\sqrt{c^2+3}+c\sqrt{a^2+3}=\sum\sqrt{a}\sqrt{ab^2+3a}\le \sqrt{(a+b+c)(ab^2+bc^2+ca^2+3(a+b+c))}$$

Ta lại có $ a^3+ac^2+a^2c\ge 3ac^2$, lập luận tương tự và cộng vế ta có:

$ 3(ab^2+bc^2+ca^2)\le (a+b+c)(a^2+b^2+c^2)=3(a+b+c)\Leftrightarrow ab^2+bc^2+ca^2\le a+b+c$, hay $Q\le 2(a+b+c) (***)$

Từ (**) và (***), ta suy ra được $VT\ge\dfrac{1}{2(a+b+c)}$. Do đó phép chứng minh hoàn tất.




#742277 $\frac{xy}{z}+\frac{yz}{x...

Gửi bởi hanguyen445 trong 28-11-2023 - 20:10

BÀI TOÁN 4.

Cho các số thực dương $a, b, c$. Chứng minh rằng $$ \frac{a}{(b+c)^2}+\frac{b}{(c+a)^2}+\frac{c}{(a+b)^2} \geq \frac{9}{4(a+b+c)}.$$

Biến đổi, kết hợp với sử dụng BĐT cauchy-schwarz, ta có $$\sum\frac{a}{(b+c)^2}=\sum\dfrac{(\dfrac{a}{b+c})^2}{a}\ge\frac{1}{a+b+c}(\sum\frac{a}{b+c})^2 (1)$$

Hơn nữa ta có $$\sum\frac{a}{b+c}=\sum\frac{a^2}{ab+ac}\ge\frac{(a+b+c)^2}{2(ab+bc+ac)}\ge\frac{3(ab+bc+ac)}{2(ab+bc+ac)} (2)$$

Từ (1) và (2) ta suy ra được $\sum\frac{a}{(b+c)^2}\ge\frac{9}{a+b+c}$. Phép chứng minh hoàn tất




#742259 $\frac{xy}{z}+\frac{yz}{x...

Gửi bởi hanguyen445 trong 27-11-2023 - 11:35

Bài toán 3. Cho các số dương $a, b, c$ thoả mãn điều kiện $a+b+c=3$. Chứng minh bất đẳng thức $$\frac{a}{1+b^2}+\frac{b}{1+c^2}+\frac{c}{1+a^2} \geq \frac{3}{2}.$$

Ta có $\frac{a}{1+b^2}=\frac{a(1+b^2)-ab^2}{1+b^2}=a-\dfrac{ab^2}{1+b^2}\ge a-\dfrac{ab}{2}$. Lập luận tương tự và cộng vế ta có

$$VT\ge (a+b+c)-\dfrac{ab+bc+ac}{2}\ge 3-\dfrac{(a+b+c)^2}{6}=\frac{3}{2}$$

Phép chứng minh hoàn tất.




#742245 $\frac{xy}{z}+\frac{yz}{x...

Gửi bởi hanguyen445 trong 26-11-2023 - 21:41

Bài toán 2. Chứng minh rằng với mọi $a, b, c, d$ dương ta luôn có $$16(abc+bcd+cda+dab) \leq (a+b+c+d)^3.$$

 

Ta có $16(abc+bcd+cda+dab) =16bc(a+d)+16ad(b+c)\leq 4(b+c)^2(a+d)+4(b+c)(a+d)^2=4(a+d)(b+c)(a+b+c+d)$

Lại có $4(a+d)(b+c)\le (a+b+c+d)^2$

 

Do đó ta suy ra được $16(abc+bcd+cda+dab) \le (a+b+c+d)^3$. 

Vậy phép chứng minh hoàn tất.